Mathcenter Forum  

Go Back   Mathcenter Forum > คณิตศาสตร์โอลิมปิก และอุดมศึกษา > คณิตศาสตร์อุดมศึกษา
สมัครสมาชิก คู่มือการใช้ รายชื่อสมาชิก ปฏิทิน ข้อความวันนี้

ตั้งหัวข้อใหม่ Reply
 
เครื่องมือของหัวข้อ ค้นหาในหัวข้อนี้
  #1  
Old 11 พฤศจิกายน 2008, 17:03
beginner01 beginner01 ไม่อยู่ในระบบ
จอมยุทธ์หน้าหยก
 
วันที่สมัครสมาชิก: 15 กันยายน 2008
ข้อความ: 177
beginner01 is on a distinguished road
Default ลิมิต

นิยาม $\displaystyle A_a(n)=\sum_{i = 1}^{n} (2i-1)^a$
$\displaystyle B_a(n)=\sum_{i = 1}^{n} (2i)^a$
$\displaystyle S_a(n)=\frac{A_a(n)}{B_a(n)}$
จงหาค่าของ $\displaystyle\lim_{n \to \infty} S_a(n)$ สำหรับแต่ละ $a\in\mathbb{R}$

ป.ล.โจทย์ดั้งเดิมคือ $\displaystyle\lim_{n \to \infty}S_{\frac{1}{7}}(n)$
ถ้าหาด้านบนไม่ได้ แต่หาอันด้านล่างได้ ก็ช่วยให้ solution ให้ทีครับ ขอบคุณครับ
__________________
จะคิดเลขก็ติดขัด จะคิดรักก็ติดพัน
ตอบพร้อมอ้างอิงข้อความนี้
  #2  
Old 11 พฤศจิกายน 2008, 21:52
หยินหยาง's Avatar
หยินหยาง หยินหยาง ไม่อยู่ในระบบ
กระบี่จักรวาล
 
วันที่สมัครสมาชิก: 06 มกราคม 2007
ข้อความ: 2,921
หยินหยาง is on a distinguished road
Default

อ้างอิง:
ข้อความเดิมเขียนโดยคุณ beginner01 View Post
นิยาม $\displaystyle A_a(n)=\sum_{i = 1}^{n} (2i-1)^a$
$\displaystyle B_a(n)=\sum_{i = 1}^{n} (2i)^a$
$\displaystyle S_a(n)=\frac{A_a(n)}{B_a(n)}$
จงหาค่าของ $\displaystyle\lim_{n \to \infty} S_a(n)$ สำหรับแต่ละ $a\in\mathbb{R}$

ป.ล.โจทย์ดั้งเดิมคือ $\displaystyle\lim_{n \to \infty}S_{\frac{1}{7}}(n)$
ถ้าหาด้านบนไม่ได้ แต่หาอันด้านล่างได้ ก็ช่วยให้ solution ให้ทีครับ ขอบคุณครับ
เคยแต่เจอโจทย์ $\displaystyle\lim_{n \to \infty}S_{-7}(n) $ ครับ
ตอบพร้อมอ้างอิงข้อความนี้
  #3  
Old 11 พฤศจิกายน 2008, 22:27
beginner01 beginner01 ไม่อยู่ในระบบ
จอมยุทธ์หน้าหยก
 
วันที่สมัครสมาชิก: 15 กันยายน 2008
ข้อความ: 177
beginner01 is on a distinguished road
Default

คือทางนี้รู้ว่าถ้า $a<-1$ ให้ b=-a แล้วได้ว่า $b>1$ และ
$\displaystyle\lim_{n \to \infty}S_a(n)=\frac{\frac{1}{1^b}+\frac{1}{3^b}+\cdots}{\frac{1}{2^b}+\frac{1}{4^b}+\cdots}$
สังเกตว่า $\displaystyle\frac{1}{1^b}+\frac{1}{3^b}+\cdots<\displaystyle\frac{1}{1^b}+\frac{1}{2^b}+\frac{1}{3^b}+\frac{1}{4^b}+\cdots$
$=\zeta(b)$
และ $\displaystyle\frac{1}{2^b}+\frac{1}{4^b}+\cdots<\displaystyle\frac{1}{1^b}+\frac{1}{2^b}+\frac{1}{3^b}+\frac{1}{4^b}+\cdots$
$=\zeta(b)$
$\therefore\displaystyle\frac{1}{1^b}+\frac{1}{3^b}+\cdots$ และ $\displaystyle\frac{1}{2^b}+\frac{1}{4^b}+\cdots$ ลู่เข้าทั้งคู่
$\displaystyle\therefore\lim_{n \to \infty}S_a(n)=\frac{(\frac{1}{1^b}+\frac{1}{3^b}+\cdots)+(\frac{1}{2^b}+\frac{1}{4^b}+\cdots)}{\frac{1}{2^b}+\frac{1}{4^b}+\cdot s}-1$
$\displaystyle =\frac{\frac{1}{1^b}+\frac{1}{2^b}+\frac{1}{3^b}+\frac{1}{4^b}+\cdots}{\frac{1}{2^b}+\frac{1}{4^b}+\cdots}-1=\frac{\zeta(b)}{\frac{1}{2^b}\zeta(b)}-1=\frac{1}{2^a}-1$
ก็คือ $\displaystyle\lim_{n \to \infty}S_a(n)=\frac{1}{2^a}-1$ เมื่อ $a<-1$
__________________
จะคิดเลขก็ติดขัด จะคิดรักก็ติดพัน

11 พฤศจิกายน 2008 22:27 : ข้อความนี้ถูกแก้ไขแล้ว 1 ครั้ง, ครั้งล่าสุดโดยคุณ beginner01
ตอบพร้อมอ้างอิงข้อความนี้
  #4  
Old 12 พฤศจิกายน 2008, 09:28
kheerae's Avatar
kheerae kheerae ไม่อยู่ในระบบ
ลมปราณบริสุทธิ์
 
วันที่สมัครสมาชิก: 07 สิงหาคม 2008
ข้อความ: 117
kheerae is on a distinguished road
Default

อ้างอิง:
ข้อความเดิมเขียนโดยคุณ beginner01 View Post
นิยาม $\displaystyle A_a(n)=\sum_{i = 1}^{n} (2i-1)^a$
$\displaystyle B_a(n)=\sum_{i = 1}^{n} (2i)^a$
$\displaystyle S_a(n)=\frac{A_a(n)}{B_a(n)}$
จงหาค่าของ $\displaystyle\lim_{n \to \infty} S_a(n)$ สำหรับแต่ละ $a\in\mathbb{R}$

ป.ล.โจทย์ดั้งเดิมคือ $\displaystyle\lim_{n \to \infty}S_{\frac{1}{7}}(n)$
ถ้าหาด้านบนไม่ได้ แต่หาอันด้านล่างได้ ก็ช่วยให้ solution ให้ทีครับ ขอบคุณครับ
แต่ผมมีวิธีคิดอีกแบบหนึ่งซึ่งไม่แน่ใจว่าจะถูกหรือป่าว ยังไงก็ช่วยชี้แนะด้วยนะครับ

$\displaystyle A_a(n)=\sum_{i = 1}^{n} (2i-1)^a = 1 + 3^a + 5^a + ... + (2n-1)^a$

$\displaystyle B_a(n)=\sum_{i = 1}^{n} (2i)^a = 2^a + 4^a + 6^a + ... + (2n)^a$

$\displaystyle\lim_{n \to \infty} S_a(n) = \lim_{n \to \infty} \frac{1 + 3^a + 5^a + ... + (2n-1)^a}{2^a + 4^a + 6^a + ... + (2n)^a}$

$\displaystyle\lim_{n \to \infty} S_a(n) = \lim_{n \to \infty} \frac{1 + 3^a + 5^a + ... + (2n-1)^a}{2^a + 4^a + 6^a + ... + (2n)^a} \times \frac{n^a}{n^a}$

$\displaystyle\lim_{n \to \infty} S_a(n) = \lim_{n \to \infty} \frac{\frac{1 + 3^a + 5^a + ... + (2n-1)^a}{n^a}}{\frac{2^a + 4^a + 6^a + ... + (2n)^a}{n^a}}$

$\displaystyle\lim_{n \to \infty} S_a(n) = \lim_{n \to \infty} \frac{\frac{1 + 3^a + 5^a + ... +}{n^a} \frac{(2n-1)^a}{n^a}}{\frac{2^a + 4^a + 6^a + ... +}{n^a} \frac{(2n)^a}{n^a}}$

$\displaystyle\lim_{n \to \infty} S_a(n) = \lim_{n \to \infty} \frac{\frac{1 + 3^a + 5^a + ... +}{n^a} (2 - \frac{1}{n^a})^a}{\frac{2^a + 4^a + 6^a + ... +}{n^a} (2)^a}$

เมื่อ take limit จะได้

$\displaystyle\lim_{n \to \infty} S_a(n) = \frac{0 + 0 + 0 + ... + (2-0)^a}{0 + 0 + 0 + ... + (2)^a}$

$\displaystyle\lim_{n \to \infty} S_a(n) = 1 , a\in \mathbb{R} $

ผมไม่แน่ใจว่าถูกหรือป่าวนะครับ ยังไงก็ช่วยชี้แนะด้วย
__________________
"ไม่มีอะไรดีไปกว่าการที่ได้ตื่นขึ้นมาอีกวัน" ผมเชื่อในปาฏิหารย์แต่ผมไม่เชื่อว่าปาฏิหารย์จะเกิดขึ้นถ้าผมไม่ทำ
ตอบพร้อมอ้างอิงข้อความนี้
  #5  
Old 12 พฤศจิกายน 2008, 11:39
nooonuii nooonuii ไม่อยู่ในระบบ
ผู้พิทักษ์กฎทั่วไป
 
วันที่สมัครสมาชิก: 25 พฤษภาคม 2001
ข้อความ: 6,408
nooonuii is on a distinguished road
Default

ถ้า $a\geq -1$ ผมได้ลิมิตเท่ากับ $1$ ครับ แต่ของคุณ Kheerae ยังมีปัญหาตรงหาลิมิตในรูป

$\dfrac{1}{n^a}+\dfrac{3^a}{n^a}+\cdots\dfrac{(2n-1)^a}{n^a}$

เพราะว่าตัวผลบวกขึ้นกับ $n$ จึงกระจายลิมิตทีละเทอมไม่ได้
__________________
site:mathcenter.net คำค้น
ตอบพร้อมอ้างอิงข้อความนี้
  #6  
Old 12 พฤศจิกายน 2008, 17:03
beginner01 beginner01 ไม่อยู่ในระบบ
จอมยุทธ์หน้าหยก
 
วันที่สมัครสมาชิก: 15 กันยายน 2008
ข้อความ: 177
beginner01 is on a distinguished road
Default

อ้างอิง:
ข้อความเดิมเขียนโดยคุณ nooonuii View Post
ถ้า $a\geq -1$ ผมได้ลิมิตเท่ากับ $1$ ครับ...
ช่วยแสดงวิธีคิดให้ดูได้ไหมครับ
__________________
จะคิดเลขก็ติดขัด จะคิดรักก็ติดพัน
ตอบพร้อมอ้างอิงข้อความนี้
  #7  
Old 12 พฤศจิกายน 2008, 20:52
Timestopper_STG's Avatar
Timestopper_STG Timestopper_STG ไม่อยู่ในระบบ
ลมปราณคุ้มครองร่าง
 
วันที่สมัครสมาชิก: 22 มกราคม 2006
ข้อความ: 256
Timestopper_STG is on a distinguished road
Send a message via MSN to Timestopper_STG
Default

I also solved it, but just for all real ${\alpha}\geq 0$...

At first, when $\alpha=0$, then we have $S_{\alpha}(n)=1,\forall n\in\mathbb{N}$

Consider $\alpha >0$.

Notice that in this case $\displaystyle{A_{\alpha}(n)<B_{\alpha}(n),\forall n\in\mathbb{N}}$.

Hence, it follows that

$$1-\frac{(2n)^{\alpha}}{B_{\alpha}(n)}=\frac{0^{\alpha}+2^{\alpha}+\cdots+(2n-2)^{\alpha}}{B_{\alpha}(n)}<S_{\alpha}(n)=\frac{A_{\alpha}(n)}{B_{\alpha}(n)}<\frac{2^{\alpha}+4^{\alpha}+\cdots+(2n)^{\alpha}}{ B_{\alpha}(n)}=1$$

Then observe that

$$\lim_{n \rightarrow \infty} \frac{B_{\alpha}(n)}{(2n)^{\alpha}}= \lim_{ n \rightarrow \infty}\frac{1^{\alpha}+2^{\alpha}+\cdots + n^{\alpha}}{n^{\alpha}}=\lim_{n \rightarrow \infty}n\left[ \frac{1}{n}\sum_{k=1}^{n}f\left( \frac{k}{n} \right) \right]$$

So look at the limit of the first inequality we have $\displaystyle{\lim_{n\rightarrow\infty}S_{\alpha}(n)=1}$.
__________________
$$\int_{0}^{\frac{\pi}{2}}\frac{a\cos x-b\sin x}{a\sin x+b\cos x}dx=\ln\left(\frac{a}{b}\right)$$
BUT
$$\int_{0}^{\frac{\pi}{2}}\frac{a\cos x+b\sin x}{a\sin x+b\cos x}dx=\frac{\pi ab}{a^{2}+b^{2}}+\frac{a^{2}-b^{2}}{a^{2}+b^{2}}\ln\left(\frac{a}{b}\right)$$

12 พฤศจิกายน 2008 20:55 : ข้อความนี้ถูกแก้ไขแล้ว 1 ครั้ง, ครั้งล่าสุดโดยคุณ Timestopper_STG
ตอบพร้อมอ้างอิงข้อความนี้
  #8  
Old 12 พฤศจิกายน 2008, 22:10
beginner01 beginner01 ไม่อยู่ในระบบ
จอมยุทธ์หน้าหยก
 
วันที่สมัครสมาชิก: 15 กันยายน 2008
ข้อความ: 177
beginner01 is on a distinguished road
Default

อ้างอิง:
ข้อความเดิมเขียนโดยคุณ Timestopper_STG View Post
$$\lim_{n \rightarrow \infty} \frac{B_{\alpha}(n)}{(2n)^{\alpha}}= \lim_{ n \rightarrow \infty}\frac{1^{\alpha}+2^{\alpha}+\cdots + n^{\alpha}}{n^{\alpha}}=\lim_{n \rightarrow \infty}n\left[ \frac{1}{n}\sum_{k=1}^{n}f\left( \frac{k}{n} \right) \right]$$
$f(\frac{k}{n})$ นี่คือฟังก์ชันอะไรครับ? แล้วก็ยังงงๆอยู่ว่าทำไม $$\lim_{n \rightarrow \infty} \frac{B_{\alpha}(n)}{(2n)^{\alpha}}=0$$
ขอขอบคุณมาล่วงหน้าเลยละกันครับ
__________________
จะคิดเลขก็ติดขัด จะคิดรักก็ติดพัน

12 พฤศจิกายน 2008 22:11 : ข้อความนี้ถูกแก้ไขแล้ว 1 ครั้ง, ครั้งล่าสุดโดยคุณ beginner01
ตอบพร้อมอ้างอิงข้อความนี้
  #9  
Old 12 พฤศจิกายน 2008, 23:19
Timestopper_STG's Avatar
Timestopper_STG Timestopper_STG ไม่อยู่ในระบบ
ลมปราณคุ้มครองร่าง
 
วันที่สมัครสมาชิก: 22 มกราคม 2006
ข้อความ: 256
Timestopper_STG is on a distinguished road
Send a message via MSN to Timestopper_STG
Default

สิ่งที่ผมต้องการจะแสดงคือ $\displaystyle{\lim_{n\rightarrow\infty}\frac{(2n)^{\alpha}}{B_{\alpha}(n)}=0}$

โดยที่เรารู้ว่า $\displaystyle{\int_{0}^{1}f(x)dx=\lim_{n\rightarrow\infty}\frac{1}{n}\sum_{k=1}^{n}f\left(\frac{k}{n}\right)}$

ดังนั้นในข้อนี้ $f(x)=x^{\alpha}$ ซึ่งทำให้ได้ผลตามต้องการ...หายงงไหมครับ
__________________
$$\int_{0}^{\frac{\pi}{2}}\frac{a\cos x-b\sin x}{a\sin x+b\cos x}dx=\ln\left(\frac{a}{b}\right)$$
BUT
$$\int_{0}^{\frac{\pi}{2}}\frac{a\cos x+b\sin x}{a\sin x+b\cos x}dx=\frac{\pi ab}{a^{2}+b^{2}}+\frac{a^{2}-b^{2}}{a^{2}+b^{2}}\ln\left(\frac{a}{b}\right)$$
ตอบพร้อมอ้างอิงข้อความนี้
  #10  
Old 12 พฤศจิกายน 2008, 23:51
beginner01 beginner01 ไม่อยู่ในระบบ
จอมยุทธ์หน้าหยก
 
วันที่สมัครสมาชิก: 15 กันยายน 2008
ข้อความ: 177
beginner01 is on a distinguished road
Default

อ้อ... เข้าใจแล้วครับ ขอบคุณมากครับ
__________________
จะคิดเลขก็ติดขัด จะคิดรักก็ติดพัน
ตอบพร้อมอ้างอิงข้อความนี้
  #11  
Old 13 พฤศจิกายน 2008, 10:54
nooonuii nooonuii ไม่อยู่ในระบบ
ผู้พิทักษ์กฎทั่วไป
 
วันที่สมัครสมาชิก: 25 พฤษภาคม 2001
ข้อความ: 6,408
nooonuii is on a distinguished road
Default

ขอเรียกย่อๆว่า $A_n,B_n$ นะครับ

case 1 $a>-1$ ให้ $f(x)=x^a$

$\displaystyle{\dfrac{2A_n}{n^{a+1}}=\dfrac{2}{n}\sum_{i=1}^n\Big(\dfrac{2i-1}{n}\Big)^a}$

$~~~~~~~~\displaystyle{=\dfrac{2}{n}\sum_{i=1}^nf(2i-1)}$

$~~~~~~~~\displaystyle{\to \int_0^2x^adx}$

$~~~~~~~~\displaystyle{=\dfrac{2^{a+1}}{a+1}}$ as $n\to\infty$

$\displaystyle{\dfrac{B_n}{n^{a+1}}=\dfrac{2^a}{n}\sum_{i=1}^n\Big(\dfrac{i}{n}\Big)^a}$

$~~~~~~~~\displaystyle{=2^a\cdot \dfrac{1}{n}\sum_{i=1}^nf\Big(\dfrac{i}{n}\Big)}$

$~~~~~~~~\displaystyle{\to 2^a\int_0^1x^adx}$

$~~~~~~~~=\dfrac{2^a}{a+1}$ as $n\to\infty$

ดังนั้น

$\displaystyle{\lim_{n\to\infty}\frac{A_n}{B_n}=1}$

case 2 $a=-1$

จากเอกลักษณ์

$1-\dfrac{1}{2}+\dfrac{1}{3}-\dfrac{1}{4}+\cdots+\dfrac{1}{2n-1}-\dfrac{1}{2n}=\dfrac{1}{n+1}+\dfrac{1}{n+2}+\cdots+\dfrac{1}{2n}$

เราจะได้ว่า

$\dfrac{A_n}{B_n}=1+\dfrac{(n+1)^{-1}+(n+2)^{-1}+\cdots+(2n)^{-1}}{B_n}$

พิสูจน์ได้ไม่ยากว่า

$\dfrac{1}{n+1}+\dfrac{1}{n+2}+\cdots +\dfrac{1}{2n}\leq \dfrac{n}{n+1}<1$


และ $B_n\to\infty$ as $n\to\infty$

ดังนั้น

$\displaystyle{\lim_{n\to\infty}\frac{A_n}{B_n}=1+0=1}$

หมายเหตุ หากต้องการลิมิตแบบละเอียดก็ใช้ Riemann sum หาได้ครับ

ให้ $g(x)=\dfrac{1}{1+x}$
$\dfrac{1}{n+1}+\dfrac{1}{n+2}+\cdots +\dfrac{1}{2n}=\dfrac{1}{n}\Big[\dfrac{1}{1+\frac{1}{n}}+\dfrac{1}{1+\frac{2}{n}}+\cdots+\dfrac{1}{1+\frac{n}{n}}\Big]$

$~~~~~~~~~~~~~~~~~~~~~~~~~~~~~~~~~~~\displaystyle{=\dfrac{1}{n}\sum_{i=1}^{n}g\Big(\dfrac{i}{n}\Big)}$

$~~~~~~~~~~~~~~~~~~~~~~~~~~~~~~~~~~~\displaystyle{\to\int_0^1\dfrac{1}{1+x}dx}$

$~~~~~~~~~~~~~~~~~~~~~~~~~~~~~~~~~~~=\ln{2}$
__________________
site:mathcenter.net คำค้น
ตอบพร้อมอ้างอิงข้อความนี้
  #12  
Old 13 พฤศจิกายน 2008, 18:14
beginner01 beginner01 ไม่อยู่ในระบบ
จอมยุทธ์หน้าหยก
 
วันที่สมัครสมาชิก: 15 กันยายน 2008
ข้อความ: 177
beginner01 is on a distinguished road
Default

อ้างอิง:
ข้อความเดิมเขียนโดยคุณ nooonuii View Post
หมายเหตุ หากต้องการลิมิตแบบละเอียดก็ใช้ Riemann sum หาได้ครับ...
จริงๆไม่ต้องใช้ Riemann sum ก็ได้ครับ ก็
$$\lim_{n \to \infty}\left(\dfrac{1}{n+1}+\dfrac{1}{n+2}+\cdots +\dfrac{1}{2n}\right)=\lim_{n \to \infty} \left(1-\dfrac{1}{2}+\dfrac{1}{3}-\dfrac{1}{4}+\cdots+\dfrac{1}{2n-1}-\dfrac{1}{2n}\right)=\ln{2}$$ เพราะจาก Taylor series $$\ln{2}=1-\dfrac{1}{2}+\dfrac{1}{3}-\dfrac{1}{4}+\cdots$$

แต่ยังไงก็ขอขอบคุณสำหรับทุกๆ post ครับ
__________________
จะคิดเลขก็ติดขัด จะคิดรักก็ติดพัน

13 พฤศจิกายน 2008 18:14 : ข้อความนี้ถูกแก้ไขแล้ว 1 ครั้ง, ครั้งล่าสุดโดยคุณ beginner01
ตอบพร้อมอ้างอิงข้อความนี้
ตั้งหัวข้อใหม่ Reply



กฎการส่งข้อความ
คุณ ไม่สามารถ ตั้งหัวข้อใหม่ได้
คุณ ไม่สามารถ ตอบหัวข้อได้
คุณ ไม่สามารถ แนบไฟล์และเอกสารได้
คุณ ไม่สามารถ แก้ไขข้อความของคุณเองได้

vB code is On
Smilies are On
[IMG] code is On
HTML code is Off
ทางลัดสู่ห้อง


เวลาที่แสดงทั้งหมด เป็นเวลาที่ประเทศไทย (GMT +7) ขณะนี้เป็นเวลา 16:17


Powered by vBulletin® Copyright ©2000 - 2024, Jelsoft Enterprises Ltd.
Modified by Jetsada Karnpracha